Skip to content

Commit

Permalink
Spec(Z) is not quasi-compact
Browse files Browse the repository at this point in the history
A bit of a brain teaser. Thanks to Bhargav Bhatt
  • Loading branch information
aisejohan committed Aug 15, 2018
1 parent e0150c3 commit 20dfbcb
Showing 1 changed file with 315 additions and 0 deletions.
315 changes: 315 additions & 0 deletions examples.tex
Original file line number Diff line number Diff line change
Expand Up @@ -5844,6 +5844,321 @@ \section{Universally submersive but not V covering}



\section{The spectrum of the integers is not quasi-compact}
\label{section-canonical}

\noindent
Of course the title of this section doesn't refer to
the spectrum of the integers as a topological space,
because any spectrum is quasi-compact as a topological space
(Algebra, Lemma \ref{algebra-lemma-quasi-compact}).
No, it refers to the spectrum of the integers in the
canonical topology on the category of schemes.

\medskip\noindent
Let $U$ be a nonprincipal ultrafilter on the set $P$ of prime numbers.
For a subset $T \subset P$ we denote $T^c = P \setminus T$ the complement.
For $A \in U$ let $S_A \subset \mathbf{Z}$ be the multiplicative
subset generated by $p \in A$. Set
$$
\mathbf{Z}_A = S_A^{-1}\mathbf{Z}
$$
Observe that $\Spec(\mathbf{Z}_A) = \{(0)\} \cup A^c \subset \Spec(\mathbf{Z})$
if we think of $P$ as the set of closed points of $\Spec(\mathbf{Z})$.
If $A, B \in U$, then $A \cap B \in U$ and $A \cup B \in U$
and we have
$$
\mathbf{Z}_{A \cap B} =
\mathbf{Z}_A \times_{\mathbf{Z}_{A \cup B}} \mathbf{Z}_B
$$
(fibre product of rings). In particular, for any
integer $n$ and elements $A_1, \ldots, A_n \in U$ the morphisms
$$
\Spec(\mathbf{Z}_{A_1}) \amalg \ldots \amalg \Spec(\mathbf{Z}_{A_n})
\longrightarrow \Spec(\mathbf{Z})
$$
factors through $\Spec(\mathbf{Z}[1/p])$ for some $p$
(namely for any $p \in A_1 \cap \ldots \cap A_n$).
We conclude that the family of flat morphisms
$\{\Spec(\mathbf{Z}_A) \to \Spec(\mathbf{Z})\}_{A \in U}$ is jointly
surjective, but no finite subset is.

\medskip\noindent
For a $\mathbf{Z}$-module $M$ we set
$$
M_A = S_A^{-1}M = M \otimes_{\mathbf{Z}} \mathbf{Z}_A
$$
Claim I: for every $\mathbf{Z}$-module $M$ we have
$$
M =
\text{Equalizer}\left(
\xymatrix{
\prod\nolimits_{A \in U} M_A \ar@<1ex>[r] \ar@<-1ex>[r] &
\prod\nolimits_{A, B \in U} M_{A \cup B}
}
\right)
$$
First, assume $M$ is torsion free. Then $M_A \subset M_P$ for all $A \in U$.
Hence we see that we have to prove
$$
M = \bigcap\nolimits_{A \in U} M_A\text{ inside }M_P = M \otimes \mathbf{Q}
$$
Namely, since $U$ is nonprincipal, for any prime $p$ we have
$\{p\}^c \in U$. Also, $M_{\{p\}^c} = M_{(p)}$ is equal to the
localization at the prime $(p)$.
Thus the above is clear because already $M_{(2)} \cap M_{(3)} = M$.
Next, assume $M$ is torsion. Then we have
$$
M = \bigoplus\nolimits_{p \in P} M[p^\infty]
$$
and correspondingly we have
$$
M_A = \bigoplus\nolimits_{p \not \in A} M[p^\infty]
$$
because we are localizing at the primes in $A$. Suppose that
$(x_A) \in \prod M_A$ is in the equalizer. Denote
$x_p = x_{\{p\}^c} \in M[p^{\infty}]$. Then the equalizer
property says
$$
x_A = (x_p)_{p \not \in A}
$$
and in particular it says that $x_p$ is zero for all but a finite
number of $p \not \in A$. To finish the proof in the torsion case
it suffices to show that $x_p$ is zero for all but a finite number
of primes $p$. If not write $\{p \in P \mid x_p \not = 0\} = T \amalg T'$
as the disjoint union of two infinite sets. Then either
$T \not \in U$ or $T' \not \in U$ because $U$ is an ultrafilter
(namely if both $T, T'$ are in $U$ then $U$ contains $T \cap T' = \emptyset$
which is not allowed). Say $T \not \in U$. Then $T = A^c$ and this
contradicts the finiteness mentioned above.
Finally, suppose that $M$ is a general module. Then we look
at the short exact sequence
$$
0 \to M_{tors} \to M \to M/M_{tors} \to 0
$$
and we look at the following large diagram
$$
\xymatrix{
M_{tors} \ar[r] \ar[d] &
\prod\nolimits_{A \in U} M_{tors, A} \ar@<1ex>[r] \ar@<-1ex>[r] \ar[d] &
\prod\nolimits_{A, B \in U} M_{tors, A \cup B} \ar[d] \\
M \ar[r] \ar[d] &
\prod\nolimits_{A \in U} M_A \ar@<1ex>[r] \ar@<-1ex>[r] \ar[d] &
\prod\nolimits_{A, B \in U} M_{A \cup B} \ar[d] \\
M/M_{tors} \ar[r] &
\prod\nolimits_{A \in U} (M/M_{tors})_A \ar@<1ex>[r] \ar@<-1ex>[r] &
\prod\nolimits_{A, B \in U} (M/M_{tors})_{A \cup B} \\
}
$$
Doing a diagram chase using exactness of the columns and the result
for the torsion module $M_{tors}$ and the torsion free module
$M/M_{tors}$ proving Claim I for $M$.
This gives an example of the phenomenon in the following lemma.

\begin{lemma}
\label{lemma-non-fpqc-descent}
There exists a ring $A$ and an infinite family of flat ring maps
$\{A \to A_i\}_{i \in I}$ such that for every $A$-module $M$
$$
M =
\text{Equalizer}\left(
\xymatrix{
\prod\nolimits_{i \in I} M \otimes_A A_i \ar@<1ex>[r] \ar@<-1ex>[r] &
\prod\nolimits_{i, j \in I} M \otimes_A A_i \otimes_A A_j
}
\right)
$$
but there is no finite subfamily where the same thing is true.
\end{lemma}

\begin{proof}
See discussion above.
\end{proof}

\noindent
We continue working with our nonprincipal ultrafilter $U$ on the set $P$
of prime numners.
Let $R$ be a ring. Denote $R_A = S_A^{-1}R = R \otimes \mathbf{Z}_A$
for $A \in U$.
Claim II: given closed subsets $T_A \subset \Spec(R_A)$, $A \in U$
such that
$$
(\Spec(R_{A \cup B}) \to \Spec(R_A))^{-1}T_A =
(\Spec(R_{A \cup B}) \to \Spec(R_B))^{-1}T_B
$$
for all $A, B \in U$, there is a closed subset $T \subset \Spec(R)$
with $T_A = (\Spec(R_A) \to \Spec(R))^{-1}(T)$ for all $A \in U$.
Let $I_A \subset R_A$ for $A \in U$ be the radical ideal cutting out $T_A$.
Then the glueing condition implies
$S_{A \cup B}^{-1}I_A = S_{A \cup B}^{-1}I_B$
in $R_{A \cup B}$ for all $A, B \in U$ (because localization preserves
being a radical ideal). Let $I' \subset R$ be the set of elements
mapping into $I_P \subset R_P = R \otimes \mathbf{Q}$.
Then we see for $A \in U$ that
\begin{enumerate}
\item $I_A \subset I'_A = S_A^{-1}I'$, and
\item $M_A = I'_A/I_A$ is a torsion module.
\end{enumerate}
Of course we obtain canonical identifications
$S_{A \cup B}^{-1}M_A = S_{A \cup B}^{-1}M_B$ for $A, B \in U$.
Decomposing the torsion modules $M_A$ into their $p$-primary
components, the reader easily shows that there exist $p$-power torsion
$R$-modules $M_p$ such that
$$
M_A = \bigoplus\nolimits_{p \not \in A} M_p
$$
compatible with the canonical identifications given above.
Setting $M = \bigoplus_{p \in P} M_p$ we find canonical isomorphisms
$M_A = S_A^{-1}M$ compatible with the above canonical identifications.
Then we get a canonical map
$$
I' \longrightarrow M
$$
of $R$-modules wich recovers the map $I_A \to M_A$ for all $A \in U$.
This is true by all the compatibilities mentioned above and the
claim proved previously that $M$ is the equalizer of the two
maps from $\prod_{A \in U} M_A$ to $\prod_{A, B \in U} M_{A \cup B}$.
Let $I = \Ker(I' \to M)$. Then $I$ is an ideal and
$T = V(I)$ is a closed subset which
recovers the closed subsets $T_A$ for all $A \in U$. This proves Claim II.

\begin{lemma}
\label{lemma-Z-not-quasi-compact}
The scheme $\Spec(\mathbf{Z})$ is not quasi-compact
in the canonical topology on the category of schemes.
\end{lemma}

\begin{proof}
With notation as above consider the family of
morphisms
$$
\mathcal{W} = \{\Spec(\mathbf{Z}_A) \to \Spec(\mathbf{Z})\}_{A \in U}
$$
By Lemma \ref{lemma-universal-effective-epimorphism-affine}
below and the two claims proved above
this is a universally effective epimorphism.
In any category with fibre products, the universal effective epimorphisms
give $\mathcal{C}$ the structure of a site (modulo some set theoretical
issues which are easy to fix) defining the canonical topology.
Thus $\mathcal{W}$ is a covering for the canonical topology.
On the other hand, we have seen above that any finite subfamily
$$
\{\Spec(\mathbf{Z}_{A_i}) \to \Spec(\mathbf{Z})\}_{i = 1, \ldots, n},\quad
n \in \mathbf{N}, A_1, \ldots, A_n \in U
$$
factors through $\Spec(\mathbf{Z}[1/p])$ for some $p$.
Hence this finite family cannot be a universally effective epimorphism
and more generally no universally effective epimorphism can refine
$\{\Spec(\mathbf{Z}_{A_i}) \to \Spec(\mathbf{Z})\}_{i = 1, \ldots, n}$.
By Sites, Definition \ref{sites-definition-quasi-compact}
this means that $\Spec(\mathbf{Z})$ is not quasi-compact
in the canonical topology. To see that our notion of quasi-compactness
agrees with the topos theoretic definition, see
Sites, Lemma \ref{sites-lemma-quasi-compact}.
\end{proof}

\begin{lemma}
\label{lemma-universal-effective-epimorphism-affine}
Let $\{f_i : X_i \to X\}_{i \in I}$ be a family of morphisms of affine schemes.
Assume that for any quasi-coherent $\mathcal{O}_X$-module $\mathcal{F}$
we have
$$
\Gamma(X, \mathcal{F}) =
\text{Equalizer}\left(
\xymatrix{
\prod\nolimits_{i \in I} \Gamma(X_i, f_i^*\mathcal{F})
\ar@<1ex>[r] \ar@<-1ex>[r] &
\prod\nolimits_{i, j \in I}
\Gamma(X_i \times_X X_j, (f_i \times f_j)^*\mathcal{F})
}
\right)
$$
Then our family of morphisms is a universal effective epimorphism
in the category of affine schemes.
\end{lemma}

\begin{proof}
Let $T$ be an affine scheme and let $h_i : X_i \to T$ be a family of morphisms
such that $h_i \circ \text{pr}_1 = h_j \circ \text{pr}_2$ on
$X_i \times_X X_j$ for all $i, j \in I$. Then
$$
\prod h_i^\sharp :
\Gamma(T, \mathcal{O}_T)
\to
\prod \Gamma(X_i, \mathcal{O}_{X_i})
$$
maps into the equalizer and we find that we get a ring map
$\Gamma(T, \mathcal{O}_T) \to \Gamma(X, \mathcal{O}_X)$
by the assumption of the lemma for $\mathcal{F} = \mathcal{O}_X$.
This ring map corresponds to a morphism $h : X \to T$ such
that $h_i = h \circ f_i$. Hence our family is an effective
epimorphism.

\medskip\noindent
Let $p : Y \to X$ be a morphism of affines. We will show
the base changes $g_i : Y_i \to Y$ of $f_i$ form an effective epimorphism
by applying the result of the previous paragraph.
Namely, if $\mathcal{G}$ is a quasi-coherent $\mathcal{O}_Y$-module, then
$$
\Gamma(Y, \mathcal{G}) = \Gamma(X, p_*\mathcal{G}),\quad
\Gamma(Y_i, g_i^*\mathcal{G}) = \Gamma(X, f_i^*p_*\mathcal{G}),
$$
and
$$
\Gamma(Y_i \times_Y Y_j, (g_i \times g_j)^*\mathcal{G}) =
\Gamma(X, (f_i \times f_j)^*p_*\mathcal{G})
$$
by the trivial base change formula
(Cohomology of Schemes, Lemma \ref{coherent-lemma-affine-base-change}).
Thus we see the property of the lemma holds for the family $g_i$.
\end{proof}

\begin{lemma}
\label{lemma-universal-effective-epimorphism}
Let $\{f_i : X_i \to X\}_{i \in I}$ be a family of morphisms of affine
schemes. Assume the assumption of
Lemma \ref{lemma-universal-effective-epimorphism-affine} holds and that moreover
for any morphism of affines $Y \to X$ the map
$$
\coprod X_i \times_X Y \longrightarrow Y
$$
is a quotient map of topological spaces.
Then our family of morphisms is a universal effective epimorphism
in the category of schemes.
\end{lemma}

\begin{proof}
To check that a family of morphisms of schemes with fixed target
is an effective epimorphism, it suffices to work locally on the target.
Thus it suffices to base change our family of morphisms
by $Y \to X$ with $Y$ affine. Set $Y_i = X_i \times_X Y$.
Let $T$ be a scheme and let $h_i : Y_i \to Y$ be a family of morphisms
such that $h_i \circ \text{pr}_1 = h_j \circ \text{pr}_2$
on $Y_i \times_Y Y_j$. Note that $Y$ as a set is the coequalizer
of the two maps from $\coprod Y_i \times_Y Y_j$ to $\coprod Y_i$:
this already follows from the first condition of the lemma
when applied to skyscraper sheaves at points of $Y$; details omitted.
Hence there is a set map of underlying sets $h : Y \to T$
compatible with the maps $h_i$. By the second condition of
the lemma we see that $h$ is continuous!
Thus if $y \in Y$ and $U \subset T$ is an affine open
neighbourhood of $h(y)$, then we can find an affine open
$V \subset Y$ such that $h(V) \subset U$.
Setting $V_i = Y_i \times_Y V = X_i \times_X V$
we can use the result proved in
Lemma \ref{lemma-universal-effective-epimorphism-affine}
to show that $h|_V : V \to U \subset T$ comes from a unique
morphism of affine schemes $h_V : V \to U$ agreeing with $h_i|_{V_i}$
as morphisms of schemes for all $i$. Gleuing these $h_V$ gives a morphism
$Y \to T$ as desired. Some details omitted; please compare with
the proof of
Descent, Lemma \ref{descent-lemma-fpqc-universal-effective-epimorphisms}.
\end{proof}







\input{chapters}
Expand Down

0 comments on commit 20dfbcb

Please sign in to comment.